Difference between revisions of "1990 AHSME Problems/Problem 29"
m (→Solution 1) |
m (→Solution 1) |
||
(5 intermediate revisions by 3 users not shown) | |||
Line 10: | Line 10: | ||
== Solution 1 == | == Solution 1 == | ||
− | Notice that inclusion of the integers | + | Notice that inclusion of the integers from <math>34</math> to <math>100</math> is allowed as long as no integer between <math>11</math> and <math>33</math> inclusive is within the set. This provides a total of <math>100 - 34 + 1</math> = 67 solutions. |
− | Further | + | Further analyzation of the remaining integers between <math>1</math> and <math>10</math>, we notice that we can include all the numbers except <math>3</math> (as including <math>3</math> would force us to remove both <math>9</math> and <math>1</math>) to obtain the maximum number of <math>9</math> solutions. |
− | Thus, < | + | Thus, <math>67 + 9 = 76</math>, yielding our answer, <math>\fbox{D}</math> |
== Solution 2 == | == Solution 2 == | ||
Line 24: | Line 24: | ||
There are <math>\tfrac13(99-12)=29</math> multiples of three in that range, so there are <math>88-29=59</math> non-multiples, and <math>3+14+59=76</math>, which is <math>\fbox{D}</math> | There are <math>\tfrac13(99-12)=29</math> multiples of three in that range, so there are <math>88-29=59</math> non-multiples, and <math>3+14+59=76</math>, which is <math>\fbox{D}</math> | ||
== See also == | == See also == | ||
− | {{AHSME box|year=1990|num-b=28|num-a= | + | {{AHSME box|year=1990|num-b=28|num-a=30}} |
[[Category: Intermediate Number Theory Problems]] | [[Category: Intermediate Number Theory Problems]] | ||
{{MAA Notice}} | {{MAA Notice}} |
Latest revision as of 18:25, 25 September 2020
Contents
Problem
A subset of the integers has the property that none of its members is 3 times another. What is the largest number of members such a subset can have?
Solution 1
Notice that inclusion of the integers from to is allowed as long as no integer between and inclusive is within the set. This provides a total of = 67 solutions.
Further analyzation of the remaining integers between and , we notice that we can include all the numbers except (as including would force us to remove both and ) to obtain the maximum number of solutions.
Thus, , yielding our answer,
Solution 2
Write down in a column the elements which are indivisible by three, and then follow each one by
We can take at most elements from the first row, and at most elements from each of the next seven rows. After that we can take only from any following row. Thus the answer is the number of integers between and inclusive which are indivisible by three.
There are multiples of three in that range, so there are non-multiples, and , which is
See also
1990 AHSME (Problems • Answer Key • Resources) | ||
Preceded by Problem 28 |
Followed by Problem 30 | |
1 • 2 • 3 • 4 • 5 • 6 • 7 • 8 • 9 • 10 • 11 • 12 • 13 • 14 • 15 • 16 • 17 • 18 • 19 • 20 • 21 • 22 • 23 • 24 • 25 • 26 • 27 • 28 • 29 • 30 | ||
All AHSME Problems and Solutions |
The problems on this page are copyrighted by the Mathematical Association of America's American Mathematics Competitions.